junkxs21
junkxs21 junkxs21
  • 21-11-2022
  • Mathematics
contestada

Kenny wants to
buy a TV with a
30 inch screen. (diagonal)
The TV is 12 inches tall.
Will it fit into a cabinet
that is 16 inches wide?

Respuesta :

Otras preguntas

7) If the systolic blood pressure is normally distributed in the population with a mean=120, SD=20, X~N(120, 20) what is the probability of a random person havi
A 45-year-old man presents with extreme asthenia and weight loss. He has been suffering from celiac disease for the past 12 years. He is at the highest risk of
12) -7x+y=-19 -2x+3y=-19
Imagine you have an unknown sample that contains one of the four macromolecules. What could you do to determine if the sample contains lipids? a) Mix the sample
Explain how a rapidly moving object have the same acceleration as a slowly moving one.
Write an equation of the line containing the given point and perpendicular to the given line. ?(0?,2?); 9x+2y=3
1. (1/x^3)^2..plz help 2.. (X^2/9)^2
The tub of a washer goes into its spin-dry cycle, starting from rest and reaching an angular speed of 5.0 rev/s in 8.0 s. At this point, the person doing the la
The sum of four consecutive integers is -18. What is the greatest of these integers?
Can somebody help this is due in an hour